Problemas - Teoría de números

Problema

P1. OMM 1987. Suma de dos fracciones que dan entero

Enviado por jesus el 23 de Mayo de 2009 - 15:31.

Consideremos dos fracciones reducidas $\frac{a}{b}$ y $\frac{c}{d}$ con $ b, d>0$ . Si la suma de estas dos fracciones es un número entero entonces $b=d$.

Problema

Problema 1, ONMAS 2008

Enviado por arbiter-117 el 22 de Mayo de 2009 - 22:42.

¿Cuántos divisores cuadrados perfectos tiene el número $ 2008^{2008} $ ?

Problema

Las cervezas de Bart Simpson

Enviado por jmd el 21 de Mayo de 2009 - 12:11.

Bart Simpson cuenta, usando sus dedos de la mano derecha, las cervezas que se ha tomado su papá en la semana. Si cuenta empezando con el meñique y termina en el índice pulgar y vuelve a empezar con el meñique, y contó 777 ¿en qué dedo terminó la cuenta? (Nota: Bart solamente tiene 4 dedos. Además, hay que suponer que sabe contar hasta 777...) ¿En qué dedo terminaría si tuviese 5 dedos?
 

 

Problema

Fermat converso (en general, espurio)

Enviado por jmd el 20 de Mayo de 2009 - 23:19.

Demostrar que si $p, q$ son dos primos distintos para los cuales $a^p\equiv a \pmod{q}$ y $a^q\equiv{a} \pmod{p}$, entonces $a^{pq} \equiv a \pmod{pq}$. }

Demostrar, con este resultado, el siguiente contraejemplo para la conversa del pequeño teorema de Fermat: $2^{340} \equiv 1 \pmod{341}$ --¡pero 341 es compuesto!

Problema

Partición de un conjunto

Enviado por jmd el 19 de Mayo de 2009 - 18:00.

Encontrar todos los enteros positivos $ n $ para los cuales el conjunto $A= \{n, n+1, n+2, n+3, n+4, n+5\}$ puede particionarse en dos subconjuntos con el mismo producto de sus miembros (el producto de los números en uno de los subconjuntos es igual al producto de los números en el otro).
 

Problema

Residuo de un factorial (módulo un primo)

Enviado por jmd el 19 de Mayo de 2009 - 12:02.

Encontrar el residuo que deja 50(50!) al dividirlo entre 53.

Problema

Inverso (mod 151) de una potencia de 2

Enviado por jmd el 19 de Mayo de 2009 - 11:21.

Encontrar un número entero positivo que al multiplicarlo por $2^{145}$ y al resultado restarle 1, se obtenga un múltiplo de 151.

Problema

Expresable como combinación lineal

Enviado por jmd el 19 de Mayo de 2009 - 10:41.

Decidir (con justificación) cuál de los tres números $2007, 2008, 2009$ podría ser expresado como una combinación lineal entera de 453 y 408, es decir, en la forma $453x+408y$, con $x, y$ enteros.
 

Problema

Encontrar un residuo

Enviado por jmd el 19 de Mayo de 2009 - 10:20.

Encontrar el residuo que deja $2009^{2008}$ al dividirlo entre $9$

Problema

Clasificación de primos que dividen a un cuadrado más uno

Enviado por jesus el 17 de Mayo de 2009 - 00:19.

Demuestra que si $ p$ es un primo impar que divide a $n^2 +1$ para algún $ n$, entonces $ p$ debe ser de la forma $4k+1$, es decir, $p \equiv 1$ (mód  4).